Video hướng dẫn giải

Lựa chọn câu để xem lời giải nhanh hơn

Gieo ngẫu nhiên một con súc sắc cân đối và đồng chất hai lần.

LG a

Hãy mô tả không gian mẫu.

Phương pháp giải:

Khi gieo: mỗi con súc sắc có thể xuất hiện một trong 6 mặt tương ứng 1,2,3,4,5,6 chấm.

Mỗi phần tử của kgm là một cặp số (x,y) \((x,y \in \{ 1;2;3;4;5;6\} )\)

Lời giải chi tiết:

Phép thử \(T\) được xét là "Gieo một con súc sắc cân đối và đồng chất hai lần".

\(Ω = \left\{{(i, j) \mid i, j = 1, 2, 3, 4, 5, 6}\right\}\).

Số phần tử của không gian mẫu là \(n(Ω) = 36\).

Cách liệt kê chi tiết:

Không gian mẫu: 

\(\begin{array}{l}
\Omega = \left\{ {\left( {1;1} \right)} \right.,\left( {1;2} \right),\left( {1;3} \right),\left( {1;4} \right),\left( {1;5} \right),\left( {1;6} \right),\left( {2;1} \right),\left( {2;2} \right),\left( {2;3} \right),\left( {2;4} \right),\left( {2;5} \right),\left( {2;6} \right),\left( {3;1} \right),\left( {3;2} \right),\left( {3;3} \right),\left( {3;4} \right),\left( {3;5} \right),\left( {3;6} \right),\left( {4;1} \right),\\
\left( {4;2} \right),\left( {4;3} \right),\left( {4;4} \right),\left( {4;5} \right),\left( {4;6} \right),\left( {5;1} \right),\left( {5;2} \right),\left( {5;3} \right),\left( {5;4} \right),\left( {5;5} \right),\left( {5;6} \right),\left( {6;1} \right),\left( {6;2} \right),\left( {6;3} \right),\left( {6;4} \right),\left( {6;5} \right),\left. {\left( {6;6} \right)} \right\}
\end{array}\)

LG b

Xác định các biến cố sau:

A: "Tổng số chấm xuất hiện trong hai lần gieo không bé hơn \(10\)";

B: "Mặt \(5\) chấm xuất hiện ít nhất một lần".

Phương pháp giải:

Liệt kê và đếm số phần tử của biến cố A: \(n(A), n(B)\).

Lời giải chi tiết:

\(A\) \(= {(6, 4); (4, 6); (5, 5); (6, 5); (5, 6); (6, 6)}\) \( \Rightarrow n(A) = 6\)

\(B\) = \({(1, 5); (2, 5); (3, 5); (4, 5); (5, 5); (6, 5); (5, 1); (5, 2); (5, 3); (5, 4); (5, 6)}\) \( \Rightarrow n(B) = 11\).

LG c

Tính \(P(A), P(B)\).

Phương pháp giải:

+) Tính xác suất của biến cố A: \(P\left( A \right) = \dfrac{{n(A)}}{{n(Ω) }}\).

Lời giải chi tiết:

\(P(A)= \dfrac{{n\left( A \right)}}{{n\left( \Omega  \right)}}\)= \(\dfrac{6}{36}\) = \(\dfrac{1}{6}\);

\(P(B)\) \( = \dfrac{{n\left( B \right)}}{{n\left( \Omega  \right)}}\) = \(\dfrac{11}{36}\).

 soanvan.me